You are on page 1of 3

1

Normalization:
⇝According to the statistical interpretation |Ψ(𝑥, 𝑡)|2 represents the probability density for
finding the particle at point 𝑥 at time 𝑡.
⇝Normalization: The probability to find the particle must be 1
when integrating probability density over all space:
+∞
∫−∞ |Ψ(𝑥, 𝑡)|2 𝑑𝑥 = 1
⇝The time-dependent wave function is Ψ(𝑥, 𝑡). We can prove that the normalization condition
of the wave function is time independent, i.e.
𝑑 +∞
(∫ |Ψ(𝑥, 𝑡)|2 𝑑𝑥) = 0
𝑑𝑡 −∞
⇝This is the result of the mathematical feature of the Schrödinger equation.
⇝If Ψ(𝑥, 𝑡) is a solution, 𝐴Ψ(𝑥, 𝑡) is also a solution. We need to find 𝐴 such that normalization
condition is satisfied. For this to be true normalization constant ′𝐴′ must be independent of 𝑥
and 𝑡.
Question: Show that if Ψ is normalized at 𝑡 = 0 then it stays normalization for all future times,
𝑑 +∞
i.e. prove that 𝑑𝑡 (∫−∞ |Ψ(𝑥, 𝑡)|2 𝑑𝑥) = 0.
Proof:
𝑑 +∞ +∞ 𝜕 +∞ 𝜕
∫ |Ψ(𝑥, 𝑡)|2 𝑑𝑥 = ∫−∞ 𝜕𝑡 |Ψ(𝑥, 𝑡)|2 𝑑𝑥
𝑑𝑡 −∞
= ∫−∞ 𝜕𝑡
(Ψ∗ Ψ)𝑑𝑥 ⟶ (1)
𝜕 ∗ ∗ 𝜕Ψ 𝜕Ψ∗
∵ 𝜕𝑡 (Ψ Ψ) = Ψ + Ψ ⟶ (2)
𝜕𝑡 𝜕𝑡
𝜕Ψ ℏ2 𝜕 2 Ψ
The TDSE is 𝑖ℏ =− +𝑉Ψ
𝜕𝑡 2𝑚 𝜕𝑥 2
𝜕Ψ ℏ 𝜕2 Ψ 1
dividing by 𝑖ℏ: = − 𝑖2𝑚 𝜕𝑥 2 + 𝑖ℏ 𝑉Ψ
𝜕𝑡
𝜕Ψ 𝑖ℏ 𝜕2 Ψ 𝑖
= 2𝑚 𝜕𝑥 2 − ℏ 𝑉Ψ Time evolution of wave
𝜕𝑡
Take its complex conjugate. function doesn’t affect the
𝜕Ψ∗ 𝑖ℏ 𝜕2 Ψ∗ 𝑖
= − 2𝑚 + ℏ 𝑉Ψ ∗ normalization i.e. Schrödi-
𝜕𝑡 𝜕𝑥 2
Put in equation (2) nger equation doesn’t
𝜕 𝑖ℏ 𝜕2Ψ 𝑖 𝑖ℏ 𝜕 2 Ψ∗ 𝑖
(Ψ∗ Ψ) = Ψ∗ ( − ℏ 𝑉Ψ) + (− 2𝑚 + ℏ 𝑉Ψ ∗ ) Ψ affect the normalization.
𝜕𝑡 2𝑚 𝜕𝑥 2 𝜕𝑥 2
𝜕 𝑖ℏ 2 𝜕 2 Ψ∗
(Ψ∗ Ψ) = ∗𝜕 Ψ
(Ψ − Ψ)
𝜕𝑡 2𝑚 𝜕𝑥 2 𝜕𝑥 2
R.H.S can also be written as,
𝜕 𝑖ℏ 𝜕 𝜕Ψ 𝜕Ψ∗
(Ψ∗ Ψ) = (Ψ∗ 𝜕𝑥 − Ψ)
𝜕𝑡 2𝑚 𝜕𝑥 𝜕𝑥
Put in equation (1)
𝑑 +∞ 𝑖ℏ +∞ 𝜕 𝜕Ψ 𝜕Ψ∗
∫ |Ψ(𝑥, 𝑡)|2 𝑑𝑥 = 2𝑚 ∫−∞
𝑑𝑡 −∞ 𝜕𝑥
(Ψ∗ 𝜕𝑥 − 𝜕𝑥
Ψ)𝑑𝑥
𝑑 +∞ 𝑖ℏ ∗ 𝜕Ψ 𝜕Ψ∗
∫−∞
|Ψ(𝑥, 𝑡)|2 𝑑𝑥 = 2𝑚 |(Ψ − Ψ)| +∞
−∞
𝑑𝑡 𝜕𝑥 𝜕𝑥
𝑑 +∞
∫ |Ψ(𝑥, 𝑡)|2 𝑑𝑥
𝑑𝑡 −∞
= 0 [∵ Ψ(𝑥, 𝑡) ⟶ 0 𝑎𝑠 𝑥 ⟶ ±∞ otherwise wave function won’t be
normalizeable]
2
Momentum (Expectation value of momentum and momentum operator)
⇝For a particle in state Ψ, the expectation value of 𝑥 is
+∞
< 𝑥 > = ∫−∞ 𝑥|Ψ(𝑥, 𝑡)|2 𝑑𝑥.
NOTE: The expectation value is the average of repeated measurements on an ensemble of
identically prepared system, not the average of repeated measurements on the same system.
Review of integration by parts:
𝑑 𝑑𝑔 𝑑𝑓
(𝑓𝑔) = 𝑓 + 𝑑𝑥 𝑔
𝑑𝑥 𝑑𝑥
𝑏 𝑑 𝑏 𝑑𝑔 𝑏 𝑑𝑓
=> ∫𝑎 𝑑𝑥 (𝑓𝑔)𝑑𝑥 = ∫𝑎 𝑓 𝑑𝑥 𝑑𝑥 + ∫𝑎 𝑑𝑥 𝑔𝑑𝑥
𝑏 𝑑𝑔 𝑏 𝑑𝑓
=> 𝑓𝑔| 𝑎𝑏 = ∫𝑎 𝑓 𝑑𝑥 𝑑𝑥 + ∫𝑎 𝑑𝑥 𝑔𝑑𝑥 Derivative is swapped with −ve
𝑏 𝑑𝑔 𝑏 𝑑𝑓
=> ∫𝑎 𝑓 𝑑𝑥 𝑑𝑥 = − ∫𝑎 𝑑𝑥 𝑔𝑑𝑥 + 𝑓𝑔| 𝑎𝑏 sign plus a boundary term

𝑑 𝑑
Question: Find < 𝑝 > = 𝑚 𝑑𝑡 < 𝑥 > = ? < 𝑥 > tells us how the center of
𝑑𝑥
Solution: Since, probability distribution moves with time
+∞
<𝑥 > = ∫−∞ 𝑥|Ψ(𝑥, 𝑡)|2 𝑑𝑥
𝑑 +∞ 𝜕
< 𝑥 >= ∫−∞ 𝑥 𝜕𝑡 (Ψ∗ Ψ) 𝑑𝑥
𝑑𝑡
𝑑 +∞ 𝑖ℏ 𝜕 𝜕Ψ 𝜕Ψ∗
< 𝑥 >= ∫−∞ 𝑥 2𝑚 𝜕𝑥 (Ψ∗ 𝜕𝑥 − Ψ) 𝑑𝑥
𝑑𝑡 𝜕𝑥
𝑑 𝑖ℏ +∞ 𝜕 𝜕Ψ 𝜕Ψ∗
< 𝑥 >= 2𝑚 ∫−∞ 𝑥 𝜕𝑥 (Ψ∗ 𝜕𝑥 − Ψ) 𝑑𝑥
𝑑𝑡 𝜕𝑥
𝑏 𝑑𝑔 𝑏 𝑑𝑓
⟶ 𝑏𝑦 𝑝𝑎𝑟𝑡 ∵ ∫𝑎 𝑓 𝑑𝑥 𝑑𝑥 = − ∫𝑎 𝑔𝑑𝑥 + 𝑓𝑔| 𝑎𝑏
𝑑𝑥
𝑑 𝑖ℏ +∞ 𝜕𝑥 𝜕Ψ 𝜕Ψ∗ 𝑖ℏ 𝜕Ψ 𝜕Ψ∗
< 𝑥 >= − 2𝑚 ∫−∞ (Ψ ∗ 𝜕𝑥 − Ψ) 𝑑𝑥 + 𝑥 (Ψ∗ 𝜕𝑥 − Ψ)| +∞
−∞
𝑑𝑡 𝜕𝑥 𝜕𝑥 2𝑚 𝜕𝑥
𝜕Ψ 𝜕Ψ∗
∵ 𝑥 (Ψ ∗ 𝜕𝑥 − Ψ)| +∞
−∞
=0 (∵ Ψ ⟶ 0 𝑎𝑠 𝑥 ⟶ ±∞)
𝜕𝑥
𝑑 𝑖ℏ +∞ 𝜕Ψ 𝜕Ψ∗
< 𝑥 >= − 2𝑚 ∫−∞ (Ψ∗ 𝜕𝑥 − Ψ) 𝑑𝑥
𝑑𝑡 𝜕𝑥
𝑑 𝑖ℏ +∞ 𝜕Ψ +∞ 𝜕Ψ∗
< 𝑥 >= − 2𝑚 [∫−∞ (Ψ∗ 𝜕𝑥 ) 𝑑𝑥 − ∫−∞ Ψ 𝑑𝑥] ⟶ (1)
𝑑𝑡 𝜕𝑥
+∞ 𝜕Ψ∗
FOR: ∫−∞ Ψ 𝑑𝑥 Integrating this term by parts
𝜕𝑥
+∞ 𝜕Ψ∗ +∞ 𝜕Ψ
∵ ∫−∞ Ψ 𝑑𝑥 = − ∫−∞ Ψ∗ 𝜕𝑥 𝑑𝑥 + ΨΨ| +∞
−∞
𝜕𝑥
+∞ 𝜕Ψ∗ +∞ 𝜕Ψ
∫−∞ Ψ 𝑑𝑥 = − ∫−∞ Ψ∗ 𝜕𝑥 𝑑𝑥 + 0
𝜕𝑥
+∞ 𝜕Ψ∗ +∞ 𝜕Ψ
∫−∞ 𝜕𝑥 Ψ 𝑑𝑥 = − ∫−∞ Ψ∗ 𝜕𝑥 𝑑𝑥
Equation (1) becomes, This gives expectation value of the
𝑑 +∞ 𝑖ℏ 𝜕Ψ +∞ 𝜕Ψ
∴ 𝑑𝑡 < 𝑥 >= − 2𝑚 [∫−∞ (Ψ ∗ 𝜕𝑥 ) 𝑑𝑥 − (− ∫−∞ Ψ∗ 𝜕𝑥 𝑑𝑥)] velocity. It’s not the same thing as
𝑑 𝑖ℏ +∞ 𝜕Ψ +∞ 𝜕Ψ
<𝑥 >= − 2𝑚 [∫−∞ Ψ∗ 𝜕𝑥 𝑑𝑥 + ∫−∞ Ψ ∗ 𝜕𝑥 𝑑𝑥] velocity of a particle. It’s not even
𝑑𝑡
𝑑 𝑖ℏ +∞ 𝜕Ψ
<𝑥 >= − 𝑚 ∫−∞ Ψ∗ 𝜕𝑥 𝑑𝑥 clear what velocity means in QM.
𝑑𝑡
3
Now,
𝑑 𝑖ℏ +∞ 𝜕Ψ
< 𝑝 > = 𝑚 𝑑𝑡 < 𝑥 > = {𝑚(− 𝑚 ∫−∞ Ψ∗ 𝜕𝑥 𝑑𝑥)} Problem: 1.7, 1.14, 1.16
+∞ 𝜕Ψ
< 𝑝 > = −𝑖ℏ ∫−∞ Ψ∗ 𝜕𝑥 𝑑𝑥
Operator that represent Momentum, In general,
+∞ 𝜕 +∞ 𝜕
<𝑝>= ∫−∞ Ψ ∗ (−𝑖ℏ 𝜕𝑥)Ψ𝑑𝑥 < 𝑄(𝑥, 𝑝) > = ∫−∞ Ψ ∗ 𝑄(𝑥, −𝑖ℏ 𝜕𝑥)Ψ𝑑𝑥

The Uncertainty Principle:


Wavelength of wavefunction Ψ is related to the momentum of the particle by de Broglie
formula:
ℎ 2𝜋ℏ
𝑝= =
𝜆 𝜆
A spread in wavelength corresponds to a spread in momentum. The more precise the particle’s
position the less precise its momentum. Quantitatively, the Heisenberg’s uncertainty principle
is

𝜎𝑥 𝜎𝑦 ≥ ,
2
Where 𝜎𝑥 is the standard deviation in 𝑥, and 𝜎𝑝 is the standard deviation in 𝑝.

You might also like